Optimal angle for a projectile from height h

In summary, the question asks for the range of a projectile shot from a height with a speed. The student tried to solve the equation for t but ran into complications. He attempted to simplify the expression but still could not find an answer. He found an easier approach using trig identities and solved for t.
  • #1
RMalayappan
16
0
Originally posted in a non-homework forum, so the homework template is missing
I'm following Professor Ramamurti Shankar's video lecture series for Fundamentals of Physics I on Open Yale and I'm hung up on the last question on the second problem set, of which there is no mention on the solution set.
http://oyc.yale.edu/physics/phys-200/lecture-4
The question asks, "Show that if a projectile is shot from a height h with speed v0 the maximum range obtains for for launch angle [tex]\theta = ArcTan(\frac{v_0}{\sqrt(v_0^2+2gh)})[/tex]

I tried this by starting with y=h+v0sinθt-1/2gt2 and x=v0cosθt. I solved for t in the y equation using the quadratic formula and got the final time tf = (v0sinθ+√(2gh+(v0sinθ)2))/g and I substituted it into the x equation and tried differentiating with respect to θ and I got some monstrosity that looks like it won't yield the answer and looks like too much work anyways for the problem. The problem is that everywhere I find this √(2gh+(v0sinθ)2) term, which looks like it should be replaced with a √(2gh+v02) term to get the intended result. Is there an easier approach that I can use to obtain the answer I'm looking for?
 
Physics news on Phys.org
  • #2
Try and simplify xf after you plug in tf by using trig identities. Reducing the amount of times theta occurs in xf will make differentiating much simplier.
 
  • #3
I have, but I still keep having to do a product rule with a square root term that ends up becoming messy.
 
  • #4
An easier start would be to use the x direction equation to express t in terms of the angle then substitute for t in the y equation.
To assist any further, we need to see your working.
 
  • Like
Likes Dr. Courtney
  • #5
I don't think I can substitute t into the y equation since I'm trying to maximize x and I need to solve for t when y=0.
Here is my work so far:
[itex]y(t)= h+v_0sin(\theta)t -\frac{1}{2}gt^2=0[/itex]

[itex]t= \frac{-v_0sin(\theta)\pm \sqrt{v_0^2sin^2(\theta) +2gh}}{-g}[/itex] and disregarding the negative solution:

[itex]t_f= \frac{v_0sin(\theta) +\sqrt{v_0^2sin^2(\theta) +2gh}}{g}[/itex]

Substituting into the x equation [itex]x=v_0cos(\theta)t[/itex]:

[itex]x_f(\theta)= v_0cos(\theta) \frac{v_0sin(\theta) +\sqrt{v_0^2sin^2(\theta) +2gh}}{g}[/itex]
Differentiating [itex]x_f[/itex] with respect to [itex]\theta[/itex] yields a complicated expression that looks like hell to solve. After simplification, it becomes:

[itex]\frac{dx_f}{d\theta} =0= \frac{v_0^2}{g} (cos(2\theta) +\frac{sin(2\theta)cos(\theta)}{\sqrt{sin^2(\theta) +\frac{2gh}{v_0^2}}} -sin(\theta) \sqrt{sin^2(\theta)+ \frac{2gh}{v_0^2}})[/itex]

and there I decided to look for other options. I'm going to try to use Lagrange multipliers to solve for the maximum of [itex]x(\theta, t)[/itex] along the constraint [itex]y(\theta, t) = 0[/itex] using [itex]\nabla x = \lambda \nabla y[/itex]
 
Last edited:
  • #6
RMalayappan said:
I don't think I can substitute t into the y equation since I'm trying to maximize x and I need to solve for t when y=0.
Why do you need to solve for t? Substitute for t as I suggested, set y=0 and differentiate wrt theta to find max x.
 
  • #7
The issue I found with that is that h drops out of the equation once y=0 with x substituted is differentiated implicitly with respect to theta. I managed to solve the equation with Lagrange multipliers with the following:

[itex]\nabla x = \lambda \nabla y[/itex] so [itex]\frac{\partial x}{\partial \theta} = \lambda \frac{\partial y}{\partial \theta}[/itex] and [itex]\frac{\partial x}{\partial t} = \lambda \frac{\partial y}{\partial t}[/itex]
From [itex]\frac{\partial x}{\partial \theta} = \lambda \frac{\partial y}{\partial \theta}[/itex], [itex]-v_0sin(\theta)t = \lambda v_0cos(\theta)t[/itex] so [itex] \lambda = -tan(\theta)[/itex]
From [itex]\frac{\partial x}{\partial t} = \lambda \frac{\partial y}{\partial t}[/itex], [itex]v_0cos(\theta) = \lambda (v_0sin(\theta)-gt)[/itex] and, by substituting [itex] \lambda [/itex] in, [itex]v_0cos(\theta) = -tan(\theta)(v_0sin(\theta)-gt)[/itex]
After some algebra: [itex]v_0(cos^2(\theta)+sin^2(\theta)) = gtsin(\theta)[/itex] so [itex]t = \frac{v_0}{gsin(\theta)}[/itex]
Substituting into y=0, simplifying and multiplying through by 2g yields [itex]0= 2gh+2v_0^2-v_0^2csc^2(\theta)[/itex]
Using the Pythagorean identity and rearranging, [itex]v_0^2cot^2(\theta)=2gh+v_0^2[/itex] so [itex]cot^2(\theta)= \frac{2gh+v_0^2}{v_0^2}[/itex]
Here the desired result is obvious: [itex]\theta = arctan(\frac{v_0}{\sqrt{2gh+v_0^2}})[/itex]
 
  • #8
RMalayappan said:
The issue I found with that is that h drops out of the equation once y=0 with x substituted is differentiated implicitly with respect to theta.
That is true, but it's ok. You still have the undifferentiated form of the equation with h in it. Both equations are true. Treat them as a pair of simultaneous equations and solve. It is no more difficult than your method above.
 

Related to Optimal angle for a projectile from height h

1. What is the optimal angle for a projectile from a given height?

The optimal angle for a projectile from a given height depends on a few factors such as the initial velocity, air resistance, and gravity. However, in most cases, the optimal angle is around 45 degrees.

2. How do you calculate the optimal angle for a projectile from a given height?

To calculate the optimal angle, you can use the equation: θ = tan^-1((2gh)/v2), where θ is the optimal angle, g is the acceleration due to gravity, h is the initial height, and v is the initial velocity.

3. Why is 45 degrees considered the optimal angle for a projectile from a given height?

45 degrees is considered the optimal angle because it allows for the maximum horizontal distance to be covered by the projectile. At this angle, the vertical and horizontal components of the initial velocity are equal, resulting in the longest possible flight time.

4. Can the optimal angle for a projectile from a given height be greater than 45 degrees?

Yes, the optimal angle can be greater than 45 degrees in some cases. This may occur when there is significant air resistance or when the initial velocity is high. In these cases, the optimal angle may be closer to 60 degrees.

5. How does air resistance affect the optimal angle for a projectile from a given height?

Air resistance can decrease the optimal angle for a projectile from a given height. This is because air resistance acts in the opposite direction of motion, slowing down the projectile and reducing its horizontal distance. This can result in a lower optimal angle, which will produce a shorter flight time and distance.

Similar threads

  • Introductory Physics Homework Help
Replies
4
Views
1K
  • Introductory Physics Homework Help
Replies
21
Views
2K
  • Introductory Physics Homework Help
Replies
11
Views
839
  • Introductory Physics Homework Help
Replies
15
Views
21K
  • Introductory Physics Homework Help
Replies
8
Views
4K
  • Introductory Physics Homework Help
Replies
3
Views
1K
  • Introductory Physics Homework Help
Replies
4
Views
2K
  • Introductory Physics Homework Help
Replies
4
Views
2K
  • Introductory Physics Homework Help
Replies
1
Views
1K
  • Introductory Physics Homework Help
Replies
1
Views
2K
Back
Top